lunes, 31 de mayo de 2010

acerca del problema del 28 de mayo

Solo para informar acerca del problema 28 de mayo que me toco proponer.
Solo se recibieron dos soluciones correctas de Irving y de Daniel, y de hecho nadie mas comento

Resultados de la centro

(Enviado por Pablo hace unas horas)

La centro fue en Mayagüez, Puerto Rico. Participaron 14 paises, ya que Barbados y Cuba no pudieron ir. Jamaica, Trinidad & Tobago e Islas Virgenes Americanas fueron los 3 paises que participaron por primera vez. Los alumnos de Mexico obtuvieron 41 puntos (Diego), 35 puntos (Julio) y 32 puntos (Fernando). Los cortes estuvieron en 36 para oro, 30 para plata y 22 para bronce. México fue el país con más puntos (108), despues estuvo Colombia con 99. Las otras dos medallas de oro fueron para dos alumnos de Colombia. En total hubo 41 estudiantes ya que Trinidad y Tobago sólo llevó a dos alumnos. Se aprobó a México para la siguiente centro y al Salvador para 2012.

Se otorgó "solución creativa" a Diego en el problema 5 y es la segunda vez que se da este reconocimiento a una persona en la centro. Se dieron 3 oros, 7 platas, 11 bronces y 10 menciones de honor. Los puntajes de los chicos de Mexico fueron
Diego: 7 7 7 7 6 7 (oro)
Julio 7 7 6 7 1 7 (plata)
Fernando 7 7 7 7 1 3 (plata)

Problema del dia 31 de mayo

En una fiesta cada invitado recibe dos sombreros uno rojo y uno azul. Al comenzar la fiesta, todos los invitados se ponen el sombrero rojo. Varias veces a lo largo de la fiesta, el locutor anuncia el nombre de alguno de los invitados y, en ese momento, el nombrado y cada uno de sus amigos se cambian el sombrero que llevan puesto por el del otro color. Demostrar que el locutor puede lograr que todos los invitados tengan puesto el sombrero azul. Dificultad 5 de IMO

Solución 30 de mayo Irving







Se cortó la imagen un poco, pero creo que si se entiende, sino me preguntan.



domingo, 30 de mayo de 2010

Problema del día: Mayo-30-10

Diez gansters están parados en un campo. La distancia entre cada par de ellos es diferente. Cuando el reloj marca exactamente las 12, cada ganster mata al ganster que tiene mas cerca. Cual es el mayor numero de ganster que puede sobrevivir.

Solución al problema propuesto por Carlos.

Mejor tarde que nunca :)

Solución 29 de mayo, Irving




sábado, 29 de mayo de 2010

Problema del día: Mayo-29-10




Desigualdad

Pongo una desigualdad que no me sale, a ver si alguien tiene idea de cómo hacerla (de preferencia sin usar multiplicadores de Lagrange).

Sean x,y,z reales positivos, demostrar que


81xyz(x^2+y^2+z^2)<=(x+y+z)^5

viernes, 28 de mayo de 2010

Solución 28 mayo, Irving

Sea n un entero mayo que 2007. Primero mostramos que podemos encontrar un triángulo de lados b_2009, r_2009 y w_2009. Podemos suponer sin pérdida de generalidad que b_2009 es el segmento más grande de los 6027 que hay en total. Entonces como b_2009 es un lado de uno de los 2009 triángulos originales tenemos b_2009< r_i+w_r para algunos enteros i,w<=2009. Pero r_i+w_r< r_2009+w_2009, entonces b_2009Voy a probar que podemos tener k=1. Sea n>2007 un entero positivo. Consideremos los siguientes 2009 triángulos: (2n,2n,n-2007) y (2t,t,t+1) con t=n,n-1,n-2,...,n-2007. En los triángulos (2t,t,t+1) pintamos de azul el lado 2t, de rojo el lado t y de blanco el lado t+1. Para el triángulo (2n,2n,n-2007) pintamos de azul un lado 2n, de rojo el otro lado 2n y de blanco el lado n-2007. Entonces para j=1,2,...,2008 se tiene b_j=2t y r_j=w_j=t para algún entero positivo t entre n y n-2007. Entonces no hay triángulo con lados b_j, r_j, w_j, excepto cuando j=2009.

Solución 27 de mayo, Irving

jueves, 27 de mayo de 2010

Problema del Día: 28 de mayo

Encuentra el mayor entero positivo k, tal que el siguiente enunciado es verdadero:

Considera 2009 triángulos no degenerados dados. En cada triángulo, los tres lados se pintan, uno de azul, uno de rojo y uno de blanco. Ahora, para cada color por separado, se ordenan las longitudes de los lados y se obtiene

b_1 <= b_2 <= ...<= b_2009 las longitudes de los lados azules
r_1 <= r_2 <= ... <=r_2009 las longitudes de los lados rojos
w_1 <= w_2 <= ...<= w_2009 las longitudes de los lados blancos

Entonces existen k indices j tales que se puede formar un triángulo no degenerado con longitudes b_j, r_j, w_j.


(problema de lista corta, equivale a un problema 1 o 4 de la IMO)

Problema 27 de mayo

Hola, Entiendo que mando este problema tarde pero técnicamente sigue siendo 27 de mayo. El problema que les mando es bastante bonito y parece que les encanta este tipo de trucos en la IMO, a ver qué les parece

Problema:
Dado p un número primo, encuentra el número de subconjuntos de { 1, 2, 3, ... , 2p } de p elementos tales que su suma sea un múltiplo de p.

Saludos desde Puerto Rico,

Pablo

Soluciones al problema del 26 y al problema de carlos

Tengo las soluciones a los dos problemas escritas en Word, pero no se como poner los dibujos y lo que escribi con ayuda del editor de ecuaciones.

Pista para el Problema del Dia 23 de mayo

Irving y Diego han solucionado el inciso a, pero nadie ha hecho el inciso b), así que les daré una pista para una de las soluciones, una solución que creo les ayudará en cuestión de una buena técnica para la IMO (y la ibero).

Supongamos que tenemos un triángulo equilátero. Sin pérdida de generalidad un vértice está en el círculo con centro en el origen. Ahora, digamos que otro vértice está en el círculo con centro en(a,b) donde a y b son enteros no negativos (sin pérdida de generalidad de nuevo).
Ahora, supongamos que los vértices son exactamente esos puntos con coordenadas enteras. Calculen donde caería el tercer vértice (hay dos opciones, escojan su favorita de las dos). Ahora demuestren que si los vértices no están en puntos con coordenadas enteras, el tercer vértice quedara muy cerca de ese punto.
Con esta información, pueden saber que tan cerca tiene que estar ese punto de un punto con coordenadas enteras. Ahora pueden usar fracciones continuas y ponerle una cota al asunto.

Lo que dice Flavio...

Hola a Todos

Espero que este post no lo consideren menos importante ya que no habla de ningun problema para resolver, y espero que no los distraiga mucho tiempo de las cosas importantes que tienen que hacer, pero hay una post que escribio Flavio que me parece bueno reproducir, esto lo escribio en respuesta a varios comentarios de un post de Jose Luis (esto por si no lo han visto):

"A mi me parece que si el le quiere dar importancia a acabar bien la escuela esta bien, por que aunque a ustedes organizadores les parece muy facil que nosotros le demos mas importancia a la olimpiada, la verdad es que la escuela tambien es bastante presion, asi que no creo que le debas como decir que salir bien de la escuela no es importante, si a el le importa, eso basta. "

Como "organizador" de la olimpiada me siento aludido directamente, y de hecho siento un "y a ustedes que les importa". Entonces creo que puedo hablar un poco en el sentir del comite.

Bueno pues resulta que ustedes 6, van a REPRESENTAR a Mexico, a todo el pais, en tal vez en el evento internacional academico mas importante del mundo, donde un resultado positivo puede marcar sus vidas de cierta manera y si esto no es suficiente para priorizar ciertas cosas entonces no entiendo nada.

Flavio si a el le importa mas la escuela (y no dijo que asi sea, es hipotetico) entonces no deberia estar en la olimpiada y punto, mucho menos ahora que esta por representar a Mexico. Creo que ustedes adquieren un gran compromiso a la hora de decidir partipar en la olimpiada, sobretodo si tienen posibilidades de estar en los primeros lugares de Mexico. Ademas se me hace ilogico que de los 500 participantes de la IMO ustedes sean los unicos que tienen presion de la escuela, entonces ahora entiendo porque nunca quedamos en el top 20.

Aqui creo que estamos hablando de que terminar bien la prepa siginifica un buen promedio, pues resulta que a nadie le va importar tu promedio de la prepa en el futuro, sin embargo una medalla de oro te abriría mucho mas las puertas que piensas abrir con un buen promedio en la prepa.

Y como "organizador" si me importa que hagan un buen papel, no por intereses personales, yo siempre he dicho que mi principal motivacion para estar en la olimpiada es para tratar de ayudar a los alumnos a que tengan las mejores condiciones para una buena participacion y creo que eso en parte puede ser la motivacion de gran parte del comite.

miércoles, 26 de mayo de 2010

solucion del prob del 24 de mayo

Me tarde en postear por que no me salia una parte (aun no me sale) pero espero entre hoy manana comentar el resto de la solucion, o que me digan si lo que estoy tratando de probar no es cierto.
Pues Si probamos que que los triangulos son homoteticos ya acabamos pues por consiguiente
seran semejantes.
pero si tomamos los triangulos de los cuales esos 3 puntos son los de tangencia del incirculo
de cada uno, entonces basta probar que esos son homoteticos.
Pero esos triangulos resultan ser ABC y el formado por las tangentes al circuncirculo de ABC
en D, E y F.
Osea basta probar que la tangente al circuncirculo por D es paralela a BC, pero eso ocurre si
y solo si D es punto medio del arco BC, que contiene a A.
Pero eso ocurre si y solo si AD es bisectriz externa de ABC, ya que esta siempre pasa por
ese punto medio del arco que contiene a A.

Solución 26 de mayo, Irving





Problema

Este es un problema de nivel IMO.

Sean a y b dos enteros postitivos que son primos relativos y tambien a-b es impar.

Si S es el conjunto con la propiedad de que a y b estan en S y que si x,y,z estan en S entonces x+y+Z esta en S.

Demuestra que todo entero mayor a 2ab esta en S

"A que país deberíamos superar en la IMO?" version 2

Acerca de la pregunta de David,
el primer comentario de Manuel ... bueno David ya le contesto, un resbalon de Manuel, tanto que hasta borro su post

Como varios han expresado que es una buena meta ganarle a Latinoamerica aqui va mi opinion.
Creo que Mexico podria ser capaz de ganarle a toda latinoamerica (basicamente a Brasil, Peru y por ahi Argentina y Colombia) mas o menos permanentemente, algo que ya sucedio en el 2006, entonces no deberia ser una meta, pues eso deberia de pasar.
Claro que lo que dice Diego de ganarle a toda America ya me parece un poco exagerado, pero una buena meta sería ganarle a Canada.

Lo que dice Carlos, espero que sea una broma

Lo que tiene mas coherencia es lo que dice Quique: a España y Portugal casi siempre se les gana, a Francia se le ha ganado tambien en ocasiones, por lo que una buena meta sería ganarles a Inglaterra e Italia.

En resumen si Mexico le gana a Canada, Inglaterra e Italia estariamos hablando de un top 20 que sería una meta excelente, esta fue mi respuesta de una preguna que hizo David ya hace tiempo.

Problema del día: 26 de mayo

Sea S un conjunto de 2004 puntos en el plano, no tres de ellos colineales. Sea L el conjunto de todas las líneas (extendidas indefinidamente en ambas direcciones) determinadas por parejas de puntos de S. Demuestra que es posible colorear los puntos de S con a lo más dos colores, tal que, para cada puntos p y q de S, el número de líneas que separan a p de q es impar sí y solo sí p y q tienen el mismo color.

martes, 25 de mayo de 2010

Post sin titulo de Jose Luis

Yo realmente no le he dedicado mucho tiempo a los problemas, y no llevo algo importante en alguno, por eso no he publicado nada. Afortunadamente ya casi salgo de la prepa y termino con los trabajos que me dejaron, para dedicarle todo el tiempo a la olimpiada.

Problema de lista corta

Este es un problema facil de una lista corta que no nos salio ni a Irving ni a mi en un determinado tiempo, a ver si les sale a ustedes.
Encontrar todas las funciones f: R-> R, tales que cumplen la ecuacion
f(f(x) + y) = 2x + f(f(y) − x) para todo reales x,y.

Lo que llevo en cada problema

He estado al pendiente del blog y he intentado todos los problemas, sin embargo no me salen, tal vez no les he dedicado lo suficiente, no he comentado sobre lo que llevo porque no se que idea sea la buena o cual poner (he notado que en los problemas o tengo 7 o tengo 1 o 0, no soy muy bueno rascandole puntos y creo que tiene que ver con eso). Pondre lo que llevo en cada problema:



19 Mayo

sea X= mcd(1,2,3,...,n) el problema se cumple si y solo si X<10^n.

sean P1,P2, ... ,Pk los primos menores o iguales a n y Pi^ai la maxima potencia de Pi que divide a algun numero menor a n para 1<=i<=k.

Notamos que Pi^ai <>
por lo tanto X=(P1^a1)(P2^a2).....(Pk^ak) <>
basta demostrar que n^k <>


Problema de polinomios NADA

Problema de prueba Ya lo habia hecho antes
llegas a que n-1<= f(n)<=n+2 y conluyes


20 mayo
Sean P1, Q1,R1 los puntos medios de BC,CA,AH1 respectivamente y M1 la proyeccion ortogonal deC1 sobre BH. Por thales R1M paralela a H1C perpendicular a AB paralela a MP1 haciendo lo mismo, llegamos a que P1MR1Q1 es un rectangulo. es conocido que H1BCA es cíclico, por brahmagupta P C1 P1, Q C1 Q1, R C1 R1, M C1 M1 son colineales respectivamente. Angulo MM1Q1=90= angulo Q1QM => QMQ1M1 es cíclico con centro en el punto medio de MQ1 que es tambien del centro del rectangulo P1MR1Q1 y analogamente, llegamos a que R1PQ1M1RP1QM es cíclico y por lo tanto M1 es el reflefajo de M on respecto al cicuncentro del triangulo PQR y esta en BH1
21 Mayo 1, 2 deben estar en el mismo subconjunto
ninguna otra cosa interesante
22 mayo
2^12 -1=(2^6+1)(2^3+1)(2^3-1)= 13*5*3*3*7 tiene 3*2*2*2=24 divisores, y por lo tato cumple, he tratado de buscar una forma de hacelo por inducción, o por contradiccion, pero aun no la encuentro algo bueno
23 mayo
no lo he intentado mucho, pero si ponemos la base del equilatero paralela al eje x y de tamaño 2n con n entero, hay que probar que existe un n tal que: n por raiz cuadrdada de 3 = k+x con
-1/500<= x <=1/500 y k entero
24 mayo y problemas de lista corta
aun no los intento

Problema 23 de mayo

Pues solo he hecho el A pero algo es algo.

Llamemosle X=(0,0), Y=(0,2k), Z=(k,RAIZ(3)*k) estos 3 puntos forman un equilatero.
Es conocido (Dirichlet's approximation theorem) que para cualquier numero real r existen infinitos p y q tales que |qr-p| < 1/q. (Se puede demostrar usando principio del palomar o tambien agarrando el desarrollo hasta n de la fraccion continua de r)

Podemos agarrar un q tal que q>1000, entonces |qRAIZ(3)-p|<1/q<1/1000, asi que agarremos k=q, y estara a menos de 1/1000 del punto W=(k,p) que es de coordenadas enteras. Asi si existe un triangulo equilatero asi (dehecho infinitos).

Sobre B, a posteriori he notado que si X pertenece al circulo (x_1,y_1), Y al (x_2,y_2) entonces Z esta en el circulo de radio 1/500 de centro G o de centro H, donde (x_1,y_1)(x_2,y_2)H y (x_1,y_1)(x_2,y_2)G son equilateros. Supongamos que es cierto.

Si XYZ es equilatero que cumple las condiciones con Z en G, tambien hay uno con Z en H asi que sin perdida de generalidad, Z esta en el circulo G. G esta a una distancia menor o igual a 3/1000 de un punto con coordenadas enteras, si y solo si hay un Z tal que Z esta a menos de 1/1000 de un punto con coordenadas enteras con XYZ equilatero. Asi que podemos trabajar con G,(x_1,y_1) y (x_2,y_2).

A cada punto le podemos restar (x_1,y_1) y si G,(x_1,y_1) y (x_2,y_2) funcionan, tambien los 3 nuevos puntos, asi que sin perdida de generalidad, (x_1,y_1)=(0,0)=O, y (x_2,y_2)=(a,b)=P. Sea el punto medio de OP,M=(a/2,b/2).Rotando B 90 grados por M contra las manecillas del reloj tenemos que B'=D+(-b/2,a/2)=((a-b)/2,(a+b)/2). Tenemos que MG/MB'= RAIZ(3), entonces G=M+RAIZ(3)(B'-D)=(a/2,b/2)+(-bRAIZ(3)/2,aRAIZ(3)/2)= ((a-bRAIZ(3))/2,(b+aRAIZ(3))/2).

Queremos que G este a menos de 3/1000 de un punto con coordenadas enteras. Sea T=(c,d) un punto entero mas cerca de G. Hasta aqui he llegado.

Solucion al problema 24 de Mayo.


A que país deberíamos superar en la IMO?

Pregunta para todos los miembros del blog, pero en especial para los 6 IMOs.

Yo creo que es un poco irreal querer superar a China  a alguna de las otras grandes potencias en la IMO, pero exactamente donde comienzan nuestras aspiraciones reales. A que país o grupo de países podríamos aspirar a superar?

Suplente

Hola a Todos

A partir de hoy se ha invitado a participar a este blog a Georges en calidad de suplente para la IMO. Como los boletos de avion a Kazhastan no estan comprados todavia, en caso de cualquier eventualidad el podria reemplazar a cualquiera de ustedes, esto incluye falta de trabajo de cualquiera de ustedes.

Saludos

Rogelio

lunes, 24 de mayo de 2010

Problema 23 mayo, algunas ideas...

Solucion al problema del 19 de mayo

Sea f(n)=[1,2,...,n]
Demostrare que f(n) menor 10^n, con lo que quedara demostrado el problema. Llamemos S al conjunto de numeros que cumple esa propiedad. Si 2k+1 esta en S,entonces tenemos que f(2k+2) menor o igual a 2*f(2k+1) menor 10*(10^(2k+1))=10^(2k+2), asi que 2k+2 esta en S.
( f(2k+2) menor o igual a 2*f(2k+1) porque la unica diferencia es el 2k+2 y si no es potencia de un primo, entonces es igual a f(2k+1), y como es par el 2k+2 solo puede ser igual a una potencia de 2, con lo que f(2k+2)=2f(2k+1) )
Es claro que f(n)=(producto de primos menores a n) (p^( pisode(log_p(n) ) )
Tenemos que
f(2n-1)/f(n)=(producto de primos menores a 2n-1)(p_i^r_i)=(Prod p menor o igual a raizde(2n-1))[p_i^r_i](Prod raizde(2n-1) menor p menor o igual a n)[p_i^r_i](Prod n menor p menor o igual a 2n-1)[p_i^r_i]
Donde p_i es el i-esimo primo de menor a mayor.

Tenemos que 0 menor o igual a r_i= PISO(log_{p_i} (2n-1) ) -PISO(log_{p_i} (n) ) menor log_{p_i} (2n-1) - (log_{p_i} (n)+1) menor log_{p_i} (2n)-log_{p_i} (n)+1=
(log_{p_i} (2)+log_{p_i} (n))-log_{p_i} (n)+1 =log_{p_i} (2)+1 menor o igual a 2, asi que r_i menor 2, y como es entero 0 menor o igual a r_i menor o igual a 1, para todo i.

Si r_i=1, y p_i menor o igual a n, entonces p_i^2 menor o igual a p_i^(PISO( log_{p_i} (n) ) + 1) menor o igual a 2n-1 y porlotanto p menor o igual a raizde(2n-1).
Porlotanto
f(2n-1)/f(n)=(Prod p menor o igual a raizde(2n-1))[p_i^r_i](Prod raizde(2n-1) menor p menor o igual a n)[p_i^r_i](Prod n menor p menor o igual a 2n-1)[p_i^r_i]
= (Prod p menor o igual a raizde(2n-1))[p_i^r_i](Prod n menor p menor o igual a 2n-1)[p_i^r_i] menor o igual a (Prod p menor o igual a raizde(2n-1))[p_i](Prod n menor p menor o igual a 2n-1)[p_i].
Es conocido que (Prod p primo menor x)[p] menor 4^x. Tenemos que para todo p primo entre n y 2n-1, p divide a (2n-1)Cn, entonces
(Prod n menor p menor o igual a 2n-1)[p_i] menor o igual a (2n-1)Cn=(2n-2)Cn+(2n-2)C(n-1) menor (1+1)^(2n-2)=4^(n-1), entonces
f(2n-1)/f(n) menor 4^(raizde(2n-1) +n-1)=e^((raizde(2n-1) +n-1)ln4)=A. Comparemoslo con B=10^(n-1)=e^((n-1)ln10)
asi que
A menor a B si y solo si (raizde(2n-1) +n-1)ln4 menor a (n-1)ln10 , y eso se cumple cuando n=7 y como el lado derecho crece mas rapido que el izquierdo, entonces se cumple para n mayor a 7 tambien.

Se comprueba facil que con n=1,2,...,10 cumple, asi que usando induccion, para todos los numeros es verdadera la desigualdad.

Problema de lista corta

Pongo un problema lista corta propuesto por Colombia, está bueno.

Considera el sistema
x+y=z+u
2xy=zu
Encuentra el máximo valor de la constante real m tal que m es menor o igual que x/y para cada solución entera positiva x, y, z, u del sistema y con x mayor o igual que y.

Solución 24 de mayo, Irving


Problema del día 24 de Mayo

Los excírculos del triángulo ABC son tangentes a los lados BC, CA y AB en A', B' y C', respectivamente. Los circuncírculos de los triángulos A'B'C, AB'C' y A'BC' intersectan al circuncírculo de ABC por segunda vez en F, D y E, respectivamente. Prueba que el triángulo DEF es semejante al triángulo formado por los puntos donde el incírculo de ABC es tangente a los lados de ABC.

Análisis de la primera semana del blog

Cada semana estaré haciendo un análisis de lo que ha pasado en el blog. Principalmente analizando el nivel se participación de ustedes 6 muchachos (inclusive les pondré una calificación).
El blog empezó el Martes 18 de Mayo, en el post "Say AYE" podemos ver que todos los IMOS ya estaban aquí para el 19 de Mayo.
El problema que puse Quique el 19 de Mayo, solo fue comentado por Irving y solo Irving presento una solución al mismo, nadie de los demás ni comento en el problema, ni comento en la solución, lo cual automáticamente les da puntos negativos, así que aquí Irving se fue adelante.
El problema de polinomios publicado por Carlos, ni fue comentado, ni fue resuelto, deben encontrar tiempo muchachos para intentar TODOS los problemas que aparezcan, y sobre todo no hay ningún pretexto para que no comenten, por mas fácil o difícil que el problema este, el mínimo comentario que debe haber es "ya lo leí y ya lo estoy intentando", para saber que están trabajando, inclusive un comentario sangrón como "ese problema esta por debajo de mi nivel y mi dignidad olímpica se vería afectada si me pongo a trabajar en el" es mejor que no comment at all. Si hay gente aquí, estudiando su doctorado, con trabajos de tiempo completo, que encuentra tiempo para ponerles problemas, no veo como ustedes estudiando prepa les sea difícil encontrarlo, especialmente gente inteligente como ustedes.
El problema de prueba publicado por Irving, solo fue comentado y resuelto por Manuel, Manuel aqui empieza a tener puntos e Irving sube todavía aun mas, pues es el primero que tiene la iniciativa de postear un problema, cualquier problema interesante que se encuentren o con el que hayan estado trabajando, nos gustaría que apareciera aquí.
El post de Isai de como escribir una solución no fue comentado por nadie, de nuevo, esperamos que comenten al menos "el link esta muy largo y no lo leí" o "yo ya me sabia todo eso" o si lo leyeron, "me pareció interesante el articulo", no esperamos que escriban un análisis profundo, pero si esperamos que lo lean y seria ideal que al menos pudieran comentar en algún punto que en particular les llamo la atención.
El problema del 20 de Mayo publicado por Eduardo, fue resuelto por Diego, Irving, Flavio y Daniel aunque solo Diego, Irving y Flavio publicaron soluciones.
La desigualdad sencilla publicada por Diego, nunca nos enteramos si en realidad era sencilla, puesto que nadie comento NADA, una vez mas, esperamos que comenten si ya la resolvieron, si ya la habían resuelto en el pasado, etc., inclusive no esperamos que publiquen soluciones completas, inclusive un esbozo en 3 renglones como "ya la hice utilizando la desigualdad útil, luego una sustitución, luego el teorema de Arztyhktlyep y listo" es mas que suficiente.
El problema de Álgebra publicado por David el 21 de Mayo, solo fue comentado por Irving y por Diego, era un problema difícil pero en el post "todavia no sale" de Irving, nos damos cuenta que el y Flavio han estado trabajando en el problema. Y hasta ahora solo Irving lo resolvió.
El problema de números publicado por Fernando el 22 de Mayo, tiene comentarios de Irving y Daniel sobre si conviene o no poner la dificultad al publicar el problema, nadie mas comento al respecto. Fue resuelto por Manuel, Diego, Irving y Flavio. Una pregunta de Quique, inclusive genero un nuevo post de Diego sobre polinimios Ciclomicos.
La pregunta de David "A que van a la IMO?" fue contestada por Diego, Manuel, Irving y Daniel. Para los 2 que faltaron, no veo ninguna razón para que no respondan preguntas de este tipo (que seguirá habiendo), ya que no les puede llevar mucho mas de 5 minutos hacerlo.

Asi que analizando únicamente lo que hicieron en el blog esta semana, estas serian mis calificaciones: A (Excelente), B (Bueno), C (Satisfactorio), D (Mediocre), F(Malo)

Irving ----- A+
Diego ------ A
Manuel ---- B
Flavio ----- B
Daniel ----- C
Jose Luis -- F




domingo, 23 de mayo de 2010

Problema del Dia 23 de Mayo

Considera el plano Cartesiano. En cada punto de coordenadas enteras (n,m), dibuja un círculo de radio 1/1000 con centro (n,m).

a) Demuestra que existe un triángulo equilátero cuyos vértices están en diferentes círculos.

b) Demuestra que cada triángulo equilátero con vértices en diferentes círculos tiene lado > 96.

Solución problema 21 de mayo, Irving

Problema 21 mayo, todavía no sale...

sábado, 22 de mayo de 2010

Polinomios ciclotomicos (Diego)

En busca de una respuesta para el problema que planteo Enrique, se me ocurrio usar polinomios ciclotomicos.(Ver links abajo)
Sea PHI_r(x) el r-esimo polinomio ciclotomico
Se tiene que
2^n-1=(multiplicacion d|n) (PHI_d(2))
y sabemos que PHI_x(2) es mayor a 1 si x es mayor a 1.
Si (con a mayor o igual a b) (PHI_a(2),PHI_b(2))=!=1 entonces a/b es una potencia de un primo.
Si n=p_1^a_1 * p_2^a_2 ... * p_m^a_m, si podemos agarrar un conjunto con P elementos tal que cada elemento del conjunto divide a n y no hay A y B (A=!=B) en el conjunto tal que A/B sea potencia de primo, entonces Tau(2^n-1) es mayor o igual a 2^P ya que habria almenos P numeros primos diferentes que dividan a 2^n-1.
Se podria agarrar el conjunto de divisores con un numero par (o impar) de factores primos diferentes. Digamos que Sigma_t es la suma de todas las posibles multiplicaciones de t-adas de primos y Sigma_t=0 si t es mayor a m. Entonces P=Sigma_1+Sigma_3+Sigma_5+...
(Llamemosle P_1 a esta suma)
o podria ser que
P=Sigma_2+Sigma_4+Sigma_6+...
(Llamemosle P_2 a esta otra)
pero tenemos que
P_1+P_2=Sigma_1+Sigma_2+...=Tau(n)-1
Asi que Max{P_1,P_2} es mayor o igual a TECHO[(P_1+P_2)/2 ]=TECHO[(Tau(n)-1)/2]
Entoncese Tau(2^n-1) es mayor o igual a 2^P que es mayor o igual a 2^TECHO[(Tau(n)-1)/2].
Bueno, no he llegado mas lejos que eso, pero creo que es un muy buen avance.



links:
Polinomio Ciclotomico

Link2

Edit (por David): Arregle los links, ahi en el Panel a la hora de hacer el post viene una opción para escribir hyperlinks

solucion prob. 22 de mayo (flavio)

Tomemos n=2^k
Entonces
2^n-1= 2^2^k - 1 = (2^(2^(k-1))+1) * (2^(2^(k-1))-1)
= ... = (2^(2^(k-1))+1) * (2^(2^(k-2))+1)*...*(2^2+1)*(2^2-1)
= (2^(2^(k-1))+1) * (2^(2^(k-2))+1)*...*(2^2+1)*(2^1+1)*(2^1-1)
= (2^(2^(k-1))+1) * (2^(2^(k-2))+1)*...*(2^2+1)*(2^1+1)*(1)
Ademas, sabemos que si a < b entonces (2^(2^a)+1,2^(2^b)+1)=1, ya que si p|2^(2^a)+1 y p|2^(2^b)+1, entonces
p|(2^(2^a)+1)*(2^(2^a)-1)=2^(2^(a+1))-1
pero como a < b entonces a+1<=b
entonces 2^(a+1) | 2^b
entonces 2^(2^(a+1))-1 | 2^(2^b)-1
entonces p|2^(2^b)-1 pero p|2^(2^b)+1
entonces
p|2^(2^b)+1-(2^(2^b)-1) = 2
pero 2^(2^b)+1 es impar, entonces (p,2)=1
entonces p|1 entonces (2^(2^a)+1,2^(2^b)+1)=1
entonces si s(r)= numero de divisores de r,
entonces
s(2^n-1)=s(2^(2^(k-1))+1) * s(2^(2^(k-2))+1)*...*s(2^2+1)*s(2^1+1)
ya que son primos relativos por parejas dos a dos.
Ahora veamos que s(i)>=2 para toda i>1, ya que 1, i son divisores distintos de i.
y que ademas hay k terminos en la multiplicacion de la derecha
(s(2^(2^0)+1), ..., S(2^(2^(k-1))+1))) y como cada uno es mayor o igual a 2, entonces su multiplicacion
es mayor o igual a 2^k=n, para ver que s(2^n-1)>n, tomemos k>5, ya que sabemo que
s(2^(2^5)+1)>2 ya que no es primo (contraejemplo a que 2^(2^n)+1 es primo siempre).
Entonces si k>5 entonces uno de esos factores sera mayor a 2 estrictamente entonces la multiplicacion
sera mayor estrictamente a 2^k=n entonces todos esos n=2^k cumplen, con k>5.

Solución 22 de mayo, Irving.

Solucion al problema de 22 de mayo (Diego)

Supongamos que n cumple, con n mayor a 1. Digamos que Tau(n) es el numero de factores de n. Tenemos que Tau(ab)=Tau(a)Tau(b) si (a,b)=1, y Tau(n) mayor o igual a 2 para todo n mayor a 1.
Tenemos que (2^n-1,2^n+1)=(2^n-1,2)=1
Tau(2^(2n)-1)=Tau(2^n-1)Tau(2^n+1) >= 2Tau(2^n-1)>2n, porlotanto 2n tambien cumple.
Haciendo eso infinitamente, encontramos una infinidad.
Tomando a n=12 que cumple, se completa la demostracion.


EDIT: Tambien podemos empezar con cualquier I impar diferente que 1.
Tenemos que
Tau(2^(2^(r+1) I)-1)=Tau(2^(2^r I)-1)Tau(2^(2^r I)+1)
Pero como 2^k+1 es primo solo cuando k es potencia de 2, y 2^r * I no lo es, entonces Tau no es primo asi que Tau(2^(2^r I)+1) mayor o igual a 3. (Dehecho solo es 3 cuando 2^(2^r I)+1 es potencia de primo, pero por Catalan eso solo se da si 2^r * I =3, osea r=0, I=3 y en el resto de los casos es minimo 4)

Asi que tenemos que la secuencia (Tau(2^I-1),Tau(2^(2I)-1),Tau(2^(4I)-1),...,Tau(2^(2^r I)-1),...) crece exponencialmente minimo a razon 3(4 para la mayoria de los casos), frente a la secuencia =I,2I,4I,...,2^r I,...) que crece siempre a razon 2, asi que para un m suficientemente grande tenemos que para todo q mayor a m
Tau(2^(2^q I)-1) > 2^q I
Con lo que hay una infinidad (para cada I>1).
Para I=1, se cumple para todo q mayor a 5, ya que Tau(2^(2^6)-1)=2^7, y la secuencia minimo crece con razon de 2, asi que siempre sera mayor de ahi en adelante.

Solucion al problema 22 de Mayo (Manuel)

Primero veamos que n=12 cumple. T(n) es el numero de divisores positivos de n. Entonces T(2^12-1)=T(4095)=T(3^2*5*7*13)=24>12, si cumple. Ahora sabemos que T(2^12 +1)>=2 ya que es un numero mayor que 1 y pues al menos es primo y al menos tiene 2 divisores. Luego multiplicando las 2 desigualdades T(2^12-1)T(2^12 +1)>2(12) entonces como T es una funcion multiplicativa y sabemos ademas que (2^12-1,2^12+1)=1 ya que son ambos impares a diferencia 2. Por lo tanto la desigualdad quedaria T((2^12-1)(2^12+1))=T(2^24-1)>24, con lo que encontramos otro n que cumple n=24. Luego haciendo lo mismo con induccion supongamos que T(2^(2^k*3)-1)>2^k*3 para alguna k, (ya hicimos el caso para k=2,3). Entonces sabemos que T(2^(2^k*3)+1)>=2 por el mismo argumento de que es mayor que 1 y al menos es primo. Ademas (2^(2^k*3)-1,2^(2^k*3)+1)=1 ya que son 2 numeros a distancia 2 impares.Entonces multiplicando ambas como T es una funcion multiplicativa tenemos que T(2^(2^k*3)-1)*T(2^(2^k*3)+1)=T(2^(2(2^k*3))-1)>2^(k+1)*3 y entonces T(2^(2^(k+1)*3)-1)>2^(k+1)*3 y entonces encontramos otro numero que cumple y terminamos la induccion entonces sabemos que T(2^(2^k*3)-1)>2^k*3 para k=2,3,... entonces encontramos una infinidad de numeros n=2^k*3 para k=2,3,... fin.

A que van a la IMO?

Para los 6 muchachos, me gustaría saber en estos momentos cuales son sus razones, motivaciones y objetivos para ir a la IMO. Exactamente a que piensan que van al concurso? No quiere que me inventen rollotes políticos que solo suenen bien sin que lo sientan realmente, en cuanto terminen de leer este post escriban inmediatamente todo lo que se les venga a la mente, sus verdaderas motivaciones, no se preocupen por lo que vayan a pensar los demás de lo que escriben, en este momento lo mas importante es que sean lo mas honestos posibles.
Si alguien ya contesto, de preferencia no lean su respuesta antes de ustedes haber publicado la suya.

Problema del dia 22 de mayo

Demostrar que hay infinitos naturales n tales que el numero de divisores positivos de 2^n -1 es mayor que n.

viernes, 21 de mayo de 2010

Solución Problema 20 mayo

Aquí está mi solución, no sé si sea igual que la que postearon ayer, pero de todas formas la pongo.

Primero, es conocido que H_1 está sobre el circuncírculo de ABC, entonces ACBH_1 es un cuadrilátero cíclico ortodiagonal. Sean M_2, M_3 y M_4 los puntos medios de BH_1, AH_1 y BC, respectivamente. Sea S el punto de intersección de MM_2 con M_3M_4. Tenemos que MM_3M_2M_4 es un rectángulo, ya que M_2M_4 y MM_3 son paralelas a CH_1 por ser los puntos medios de los lados; M_2M_3 y MM_4 son paralelas a AB, además AB y CH_1 son perpendiculares. Llamemos Z al rectángulo anteriror.

Por otra parte, como ACBH_1 es ortodiagonal se tiene que M_2C_1 es perpendicular a AC, así que M_2, C_1 y Q son colineales. De manera análoga M_4, C_1 y P son colineales, al igual que M_3, C_1 y R.

Voy ahora a mostrar que P, R y Q están sobre la circunferencia X circunscrita a Z. Los ángulos M_4RM_3 y M_4MM_3 son rectos, así que M_4RM_3M es cíclico y por lo tanto R está sobre X. De igual manera P y Q están sobre X porque PM_4MM_3 y QM_2M_3M son cíclicos. Entonces el circuncentro de PQR es S y como S es punto medio de MM_2, entonces M_2=M_1.

Problema del día: Mayo-21-10 (Álgebra)

Demostrar que no podemos partir los enteros positivos en 3 partes no vacías, de tal forma que si a y b pertenecen a partes diferentes, entonces a2 - ab + b2 pertenece a la tercera de las partes.

jueves, 20 de mayo de 2010

Recordatorio

Hola a todos

Les recuerdo que cada dia los seis tiene que intentar el problema del dia, por ejemplo el de hoy que fue del Chino y solo Daniel y Diego comentaron.

Hasta ahora falta que Flavio y Jose Luis tengan su guia, espero que los honorables miembros del comite escogan a alguno de ustedes para estarlos checando, creo que David llevara la cuenta de los que comentan y de los que no, ademas de quienes resuelven el problema primero

Desigualda ¿sencilla? (problema 2, IMO2000)

a,b,c reales positivos tales que abc=1. Demostrar que
(a-1+1/b)(b-1+1/c)(c-1+1/a)<=1

Solucion al problema del dia


Tomemos M' como el punto medio de H'B.
Llamemosle E a la proyeccion de E_1 en H'B , F al puntomedio de AH' y D al punto medio de AB.

Tenemos que (son todos angulos en esta parte)
ang CDM=ang CBA=90-ang RC_1B= ang CC_1R
Porque CRC_1Q es ciclico ya que tiene 2 angulos opuestos de 90 entonces ang CC_1R=ang CQR. Entonces ang CQR= ang CMD entonces MRDQ es ciclico, y similarmente lo son MQPF, PFEM' y EM'DR.

Como las diagonales del ACBH' son perpendiculares, entonces P,C_1 y D son coliniales al igual que RC_1F. Entonces ang DPF=90= ang DRF, entonces DPFR es ciclico. Similarmente MQEM' es ciclico.

Veamos las circunferencias MRDQ,DPFR y MQPF. El eje radical de la primera con la segunda es RD, el de la primera con la tercera es MQ y el de la segunda con la tercera es PF, pero esas 3 lineas no concurren entonces dos de esas circunferencias son la misma, porlotanto los seis puntos son conciclicos.
Haciendo lo mismo de 6 en 6 puntos obtenemos que MQPFEM'DR es ciclico.

Tenemos que como angFRD=90 entonces el punto medio de F y D es O. Por Varignon tenemos que MFM'D es paralelogramo, porlotanto el punto medio de MM' es el mismo que el de FD , que es O.

Porlotanto M' es el reflejado de M con respecto a O, y como solo hay un reflejado entonces M'=M y como M' esta sobre BH', tambien lo esta M_1 (y dehecho es su punto medio)

Problema del día: Mayo-20-10







miércoles, 19 de mayo de 2010

Como escribir una solución

Aqui les dejo un artículo muy interesante, tiene tips de como escribir soluciones. Una solución que no es clara muchas veces puede costar puntos, y esos puntos pueden ser la diferencia entre una medalla y otra. Espero les guste.

How to Write a Solution

Problema de prueba

Este problema es más que nada para ver si funciona la alternativa a LateX de subir imágenes de las fórmulas. Es de la Olimpiada de Bulgaria.

¿Existirá una función f de los naturales en los naturales tal que


para todo n >1?

Solución al problema del día

A ver si aquí si se puede...

Sean p_1, P_2,...,p_t todos los primos menores que n. Entonces t=pi(n), donde pi(x) es el la cantidad de primos menores o iguales que x. Sea r_i un entero positivo tal que
p^r_i< o=" n" i="1,2,...,t.">n ln(n)/(1.25506n log n)

=ln(n)/[1.25506 log n]= ln 10/1.25506
= 1.83464... >1

Problema de Polinomios

Problem a 1

Aquellos que esten presentes say "AYE"

Por favor pueden contestar en estos comments si ya entraron al blog para saber quien ya llego de los 6 IMO's. Puntos negativos al que llegue al ultimo, advertidos, tendremos competencias de todo tipo entre ustedes, que pueden culminar en el cumplimiento de apuestas interesantes ...........

Problema del Día 19 de Mayo

Demuestra que para todo entero positivo n existe un número de n dígitos que es múltiplo de 1,2,3,4,...,n.

martes, 18 de mayo de 2010

Guias

Hola a Todos

Cada uno de ustedes tendra un guia que revisará que esten trabajando, por ahora Daniel, Manuel e Irving ya tienen asignado un guia, que son Rogelio, David y Marco, respectivamente.
Espero que los demas tengan su guia proximamente

Rumbo a Kazajstan

Ahora que ya tenemos a la selección que nos representara en la Olimpiada Internacional de Matemáticas, nuestro objetivo es seguir trabajando duro con los muchachos para lograr la mejor participación posible en la IMO. Este blog esta dirigido principalmente a nuestros 6 alumnos, la idea es comentar problemas, soluciones, teoría y cualquier cosa que pensemos les puede ayudar a los muchachos a lograr una mejor participación. Para lograr excelencia no hay mas que trabajar, trabajar y trabajar aun mas. Con talento, mucho trabajo y una determinación indomable se pueden lograr metas extrordinarias.